LSAT and Law School Admissions Forum

Get expert LSAT preparation and law school admissions advice from PowerScore Test Preparation.

 Administrator
PowerScore Staff
  • PowerScore Staff
  • Posts: 8917
  • Joined: Feb 02, 2011
|
#81454
Complete Question Explanation

Main Point. The correct answer choice is (E).

Answer choice (A):

Answer choice (B):

Answer choice (C):

Answer choice (D):

Answer choice (E): This is the correct answer choice.

This explanation is still in progress. Please post any questions below!
 helloworld
  • Posts: 1
  • Joined: Jul 04, 2019
|
#66170
[admin note: full text of LSAT question removed from original post due to LSAC copyright regulations]

The answer is E. I don't see a better alternative, but I'm not fully convinced E is a fully correct answer either.

My thinking is that the argument only states that the food companies can't be relied upon to initiate economic changes that lead to reform. The support for the argument is the profit motive, and doesn't exclude being able to rely on the companies for reform of the food-distribution system apart from economic changes.
 George George
PowerScore Staff
  • PowerScore Staff
  • Posts: 48
  • Joined: Jun 07, 2019
|
#66199
@helloworld

Good Q! I think the key to understanding (E) and not feeling uncomfortable by the slight twist in language is to notice that the original stimulus links "economic change" with "reform[ing] the world's food-distribution system" by saying that the grain companies "cannot be relied on to initiate economic changes that would reform the world's food-distribution system."

Now, you might be tempted to think that the argument didn't rule out non-economic reforms, and so the conclusion shouldn't be that specific. (You're right that the author's argument is flawed in that way - and points to you for noticing that discrepancy!) However, Main Point Qs don't ask you for the flaw in the argument. They just ask you what the author's main conclusion is. So, even if the argument is flawed (as most LSAT arguments are), it won't change the answer to this Q-type. The answer will still be a restatement of the conclusion in the author's argument. (Personally, I often stop reading an LR stimulus on a Main Point Q after I've found the author's opinion. Reading the premises is just distracting, and makes me overthink the Q.)
User avatar
 josh1ua
  • Posts: 6
  • Joined: Jun 13, 2021
|
#88825
Is the conclusion in this stimulus the first sentence?

-Josh
User avatar
 Beatrice Brown
PowerScore Staff
  • PowerScore Staff
  • Posts: 75
  • Joined: Jun 30, 2021
|
#88887
Hi Josh!

The conclusion in this stimulus is the second half of the first sentence (beginning with "they cannot be relied on..."). The first half of the first sentence is actually a premise that supports this conclusion because it gives us a reason (maximizing profits) to believe the conclusion that we cannot rely on these companies to bring about changes to reform the food-distribution system. The rest of the stimulus advances premises that support this conclusion, describing how any actions that do causes economic changes are actually incidental and occur because of the companies' motives to maximize profit.

Since this is a Main Point question, you want to identify the conclusion of the argument and then find the answer choice that best restates this conclusion. In this case, that answer choice is answer choice (E).

I hope this helps!
 sofisofi
  • Posts: 23
  • Joined: Mar 31, 2022
|
#99334
Hi

I was wondering how the second sentence relates to the argument structure as a whole. It it a premise or an intermediate conclusion?
 Luke Haqq
PowerScore Staff
  • PowerScore Staff
  • Posts: 742
  • Joined: Apr 26, 2012
|
#99362
Hi sofisofi!

I can see how one might understand the second sentence as an intermediate conclusion. It's important to make a note about your question, though. You ask,

It it a premise or an intermediate conclusion?
A subsidiary conclusion is a premise. A subsidiary conclusion is a piece of the stimulus that both serves as a premise and also functions as a (subsidiary) conclusion. So a good question is: is it only a premise, or is it also an intermediate conclusion?

When I read this stimulus, the second and third sentences seemed like real-world data/instantiation of some of the variables from the first sentence. They are affirming the overall conclusion in the first sentence. The overall conclusion is that the companies cannot be relied on to initiate the mentioned change. The second and third sentences go on to explain why they cannot be relied on--namely, though the companies might bring about that change occasionally, this is incidental to their drive for maximizing profits.

In considering whether a given premise is also a subsidiary conclusion, you can ask whether there are other statements in the stimulus used in support of it. From that perspective, it seems to make sense that the third sentence supports the second. The second is stating that the mentioned change is incidental to the drive for profits, and the third sentence supports this by explaining that maximizing profits normally depends on a stable environment.

Get the most out of your LSAT Prep Plus subscription.

Analyze and track your performance with our Testing and Analytics Package.